Use your calculator to estimate the value of
log752.
Click on the correct answer.
1.450
1.653
2.031

Answers

Answer 1

The value of the given logarithm using calculator is 2.031

Logarithm functions

Logarithm are inverse of exponential functions. Given the following logarithm expression;

log(7)52

This can also be expressed as:

log(7)52 = log52/log7

log(7)52 = 0.5399/0.9804

log(7)52  = 2.031

Hence the value of the given logarithm using calculator is 2.031

Learn more on logarithm here: https://brainly.com/question/25710806

#SPJ1


Related Questions

how to find volume of a prism

Answers

Answer:

- HOW TO FIND VOLUME OF PRISM -

1st step - Write the formula for finding the volume of a regular pentagonal prism. The formula is V = [1/2 x 5 x side x apothem] x height of the prism.

2nd step - Find the area of the pentagonal base face. Let's say the length of a side is 6 cm and the length of the apothem is 7 cm.

3rd step - Find the height. Let's say the height of the shape is 10 cm.

4th step - Multiply the area of the pentagonal base face times the height. ...

and last step - State your answer in cubic units.

Step-by-step explanation:

:):):)

An American roulette wheel has 38 slots: two slots are numbered 0 and 00, and the remaining slots are numbered from 1 to 36. Find the probability that the ball lands in an odd-numbered slot. Write your answer as a whole number or reduced fraction.

Answers

Answer:

9/19

Step-by-step explanation:

To write the probability as a fraction, put the total number on the bottom and put the number of ways the event can happen, put that on top.

They said there are 38 slots. That's the total, it goes on the bottom.

Landing on an Odd Number is the event we're looking at. There are 18 ways that can happen: 1, 3, 5, 7, 9, 11, 13, 15, 17, 19, 21, 23, 25, 27, 29, 31, 33, 35

That number 18 (bc 18 ways to get an odd number) goes on top.

P(Odd) = 18/38

Always simplify when possible.

18/38

= 9/19

Consider Mary's experiment regarding whether learning of 6th graders on a math lesson is affected by background noise level. Mary has collected her data. What is the null hypothesis for her study

Answers

a) If μ₁ is the average learning rate of the 6th graders without background noise level and μ₂  is the average learning rate of the 6th graders with the background noise level

The Null Hypothesis is that the learning rate of the 6th graders is not affected by the background noise level.

The null hypothesis, H₀:  μ₁ = μ₂

b) The Alternative Hypothesis is that the learning rate of the 6th graders is affected by the background noise level.

An alternative hypothesis, Hа:  μ₁ ≠ μ₂

c) Assumptions that must be met about the data before she can correctly use an independent t-test

There must be a random selection of the 6th graders

That the two groups are normally similar in their learning abilities

The division of students into the two groups should be at random

d) She has to make these assumptions to prevent bias and inaccuracy of results. If these assumptions are not made, the outcome of the experiment may not reflect the true effect of background noise on the learning of the 6th graders.

She can still get accurate results if she includes some bias in the selection to prove a particular result.

Learn more about the null hypothesis at

https://brainly.com/question/15980493

#SPJ4

Line I is parallel to line M. If the measure of 6 is 75 degrees, what is the measure of 3?
(please look at photo attached)

Answers

In the given diagram, the measure of ∠3 will be 105°.

In the given diagram, ∠3 and ∠6 are consecutive interior angles.

How to form supplementary angles by transversal?

If two parallel lines are cut by a transversal, then the pairs of consecutive interior angles formed are supplementary.

That is,

∠3 + ∠6 = 180°

From the given information,

∠6 = 75°

Then,

∠3 + 75° = 180°

∠3 = 180° - 75°

∠3 = 105°

Hence, the measure of ∠3 will be 105°.

Learn more about the measures of angles here: brainly.com/question/2883630

#SPJ1

An approximate solution to an equation is found using this iterative process.
(x₂)³-1
and x₁ = -1
4
Xn+1 =
4/34
a) (i) Work out the value of x₂
(ii) Work out the value of x3
b) Work out the solution to 6 decimal places.

Answers

Answer:

a) i) -0.5

   ii) -0.28125

b)  -0.254102 (6 d.p.)

Step-by-step explanation:

Given iteration formula:

[tex]x_{n+1}=\dfrac{\left(x_n\right)^3-1}{4} \quad \textsf{and} \quad x_1=-1[/tex]

Part (a)(i)

Substitute the value of x₁ into the formula and solve for x₂ :

[tex]\begin{aligned}\implies x_2 & =\dfrac{\left(x_1\right)^3-1}{4}\\\\& =\dfrac{\left(-1\right)^3-1}{4}\\\\ & = \dfrac{-1-1}{4}\\\\ & = \dfrac{-2}{4}\\\\ & = -0.5\end{aligned}[/tex]

Part (a)(ii)

Substitute the value of x₂ into the formula and solve for x₃ :

[tex]\begin{aligned}\implies x_3 & =\dfrac{\left(x_2\right)^3-1}{4}\\\\& =\dfrac{\left(-0.5\right)^3-1}{4}\\\\ & = \dfrac{-0.125-1}{4}\\\\ & = \dfrac{-1.125}{4}\\\\ & = -0.28125\end{aligned}[/tex]

Part (b)

To find the solution to 6 decimal places, keep substituting each new value into the iteration formula until the answers are the same when rounded to the required level of accuracy.

[tex]\implies x_4=-0.2555618286...[/tex]

[tex]\implies x_5=-0.2541728038...[/tex]

[tex]\implies x_6=-0.2541051331...[/tex]

[tex]\implies x_7=-0.2541018552...=-0.254102\:\: \sf (6 \:d.p.)[/tex]

[tex]\implies x_8=-0.2541016964...=-0.254102\:\: \sf (6 \:d.p.)[/tex]

[tex]\implies x_9=-0.2541016888...=-0.254102\:\: \sf (6 \:d.p.)[/tex]

Therefore, the solution is -0.254102 (6 d.p.).

Let g(x) = f(x+3) for some function f(x). Is g(x) a function? From the previous problem, let the domain of f(x) be [1, 4). What is the domain of g(x)?

Answers

The domain of g(x) is [4, 7)

How to determine the domain of g(x)?

The function is given as:

g(x) = f(x + 3)

Since f(x) is a function, then g(x) is also a function

The domain of f(x) is given as:

[1, 4)

The equivalent of these values in g(x) are

x = 1 + 3 = 4

x = 4 + 3 = 7

Hence, the domain of g(x) is [4, 7)

Read more about domain at

https://brainly.com/question/1770447

#SPJ1

I don’t understand this, can somebody help?

Answers

29)[tex]quarters = 13 \\ dimes = 25 \\ nickles = 18 \\ pennies = 30 \\ total \: coins = 86[/tex]

[tex]not \: penny \: or \: dime = quarter \: or \: nickle[/tex]

[tex]p(x) = \frac{13 + 18}{86} = \frac{31}{86} = 0.36[/tex]

30)

[tex]gallons \: for \: car \: 1 = \frac{450}{18} = 25 \: gal[/tex]

[tex]gallons \: for \: car \: 2 = \frac{450}{25} = 18 \: gal[/tex]

[tex]we \: need \: 25 + 18 = 43 \: gallons[/tex]

[tex]43 \times 2.49 = 107.07 \: dollars[/tex]

A triangle is shown. One side has a length of 16, another side has a length of 20, and the length of the longest side is unknown.

Janice examines the given triangle and estimates that the longest side has a length of 25 units―if it is a right triangle. How does her estimate compare to the actual length?
It is exactly correct.
It is under by approximately 0.6 units.
It is over by approximately 0.6 units.
It is over by 13 units.

Answers

Janice estimate is under by approximately 0. 6 units. Option B

How to determine the estimate

The longest part is called the hypotenuse

The formula for finding the hypotenuse, we use the Pythagorean theorem

Hypotenuse = square root of both opposite square and adjacent square

We have that,

Hypotenuse = [tex]\sqrt{16^2 + 20^2}[/tex]

Hypotenuse = [tex]\sqrt{256+ 400}[/tex]

Hypotenuse = [tex]\sqrt{656}[/tex]

Find the square root

Hypotenuse = 25. 6

The length of the longest part = 25. 6

Janice estimated it to be 25

Comparing her estimate with the actual length = Actual length - estimate

⇒ 25. 6 - 25

⇒ 0. 6

Therefore, Janice estimate is under by approximately 0. 6 units. Option B

Learn more about Pythagorean theorem here:

https://brainly.com/question/343682

#SPJ1

Answer: B. It is under by approximately 0.6 units.

Step-by-step explanation: Just did it on edge 2023

1. How could you figure out the coordinates of a reflection over the x-axis? What would the coordinates of that reflected point be?

2. Then also tell how you could determine the coordinates when (-8,2) is reflected over the y-axis, and tell the coordinates of that new point.

Answers

The reflection of a point over the x axis is given by the equation (x, y) ⇒ (x, -y).

If the point (-8,2) is reflected over the y-axis, the new point would be (8, 2)

What is a transformation?

Transformation is the movement of a point from its initial location to a new location. Types of transformation are reflection, translation, rotation and dilation.

The reflection of a point over the x axis is given by the equation (x, y) ⇒ (x, -y).

If the point (-8,2) is reflected over the y-axis, the new point would be (8, 2)

Find out more on transformation at: https://brainly.com/question/4289712

#SPJ1

justification two common tangents

Answers

Two common tangents occur when two circles intersect each other at two points.

How to illustrate the information?

It should be noted that two common tangents are also referred to as transverse common tangents.

In this case, two common tangents occur when two circles intersect each other at two points.

The diagram is attached.

Learn more about tangent on:

brainly.com/question/4470346

#SPJ1

Find the probability for the experiment of drawing two marbles at random (without replacement) from a bag containing three green, three yellow, and four red marbles.

The marbles are different colors.

Answers

Answer:

[tex]\frac{11}{15} =0.733333333333[/tex]

Step-by-step explanation:

• Here The sample space S is the set of possible outcomes (ordered pairs of marbles) that we can draw at random (without replacement) from the bag.

Then

[tex]\text{cardS} =P^{2}_{10}=10\times 9=90[/tex]

……………………………………………

Drawing two marbles where the marbles are different colors

means

drawing (1green ,1 yellow) or (1green ,1 red) or (1yellow ,1 red)

Remark: the order intervene

=========================

•• Let E be the event “Drawing two marbles where the marbles are different colors”.

CardE = 2×3×3 + 2×3×4 + 2×3×4 = 66  (2 is for the order)

Conclusion:

[tex]p\left( E\right) =\frac{66}{90} =\frac{11}{15} =0.733333333333[/tex]

Method 2 :

[tex]p\left( E\right) =2\times \frac{3}{10} \times \frac{3}{9} +2\times \frac{3}{10} \times \frac{4}{9} +2\times \frac{3}{10} \times \frac{4}{9} =0.733333333333[/tex]

Answer:

[tex]\sf \dfrac{11}{15}[/tex]

Step-by-step explanation:

The bag of marbles contains:

3 green marbles3 yellow marbles4 red marbles

⇒ Total marbles = 3 + 3 + 4 = 10

Probability Formula

[tex]\sf Probability\:of\:an\:event\:occurring = \dfrac{Number\:of\:ways\:it\:can\:occur}{Total\:number\:of\:possible\:outcomes}[/tex]

First draw

[tex]\implies \sf P(green)=\dfrac{3}{10}[/tex]

[tex]\implies \sf P(yellow)=\dfrac{3}{10}[/tex]

[tex]\implies \sf P(red)=\dfrac{4}{10}[/tex]

Second draw

As the first marble is not replaced there are now 9 marbles in the bag.

If the first marble was green, the probability of drawing a yellow is now 3/9 and the probability of drawing a red is now 4/9.

If the first marble was yellow, the probability of drawing a green is now 3/9 and the probability of drawing a red is now 4/9.

If the first marble was red, the probability of drawing a green is now 3/9 and the probability of drawing a yellow is now 3/9.

To find the individual probabilities of picking 2 different colors, multiply the probability of the first draw by the probability of the second draw:

[tex]\implies \sf P(green)\:and\:P(red)=\dfrac{3}{10} \times \dfrac{4}{9}=\dfrac{12}{90}[/tex]

[tex]\implies \sf P(green)\:and\:P(yellow)=\dfrac{3}{10} \times \dfrac{3}{9}=\dfrac{9}{90}[/tex]

[tex]\implies \sf P(yellow)\:and\:P(green)=\dfrac{3}{10} \times \dfrac{3}{9}=\dfrac{9}{90}[/tex]

[tex]\implies \sf P(yellow)\:and\:P(red)=\dfrac{3}{10} \times \dfrac{4}{9}=\dfrac{12}{90}[/tex]

[tex]\implies \sf P(red)\:and\:P(green)=\dfrac{4}{10} \times \dfrac{3}{9}=\dfrac{12}{90}[/tex]

[tex]\implies \sf P(red)\:and\:P(yellow)=\dfrac{4}{10} \times \dfrac{3}{9}=\dfrac{12}{90}[/tex]

To find the probability of drawing two marbles at random and the marbles being different colors, add the individual probabilities listed above:

[tex]\begin{aligned}\implies \sf P(2\:different\:color\:marbles) &=\sf \dfrac{12}{90}+\dfrac{9}{90}+\dfrac{9}{90}+\dfrac{12}{90}+\dfrac{12}{90}+\dfrac{12}{90}\\\\ & = \sf \dfrac{66}{90}\\\\ & =\sf \dfrac{11}{15}\end{aligned}[/tex]

The numbering system we commonly use is called the decimal numbering system because it uses ____ symbols to represent all possible numbers.

Answers

The numbering system we commonly use is called the decimal numbering system because it uses 10 symbols to represent all possible numbers.

A way of writing numbers is known as a number system.

There are the following number systems:

Decimal number system: "Deci" meaning 10, implies that the number system consists of 10 digits or symbols, namely 0, 1, 2, 3, 4, 5, 6, 7, 8, and 9.Binary number system: "Bi" meaning 2, implies that the number system consists of 2digits or symbols, namely 0, and 1.Octal number system: "Oct" meaning 8, implies that the number system consists of 8 digits or symbols, namely 0, 1, 2, 3, 4, 5, 6, 7, and 8.Hexa-Decimal number system: "Hexa-Deci" meaning 16, implies that the number system consists of 16 symbols, namely 0, 1, 2, 3, 4, 5, 6, 7, 8, 9, A, B, C, D, E, and F.

Hence, we can say that the numbering system we commonly use is called the decimal numbering system because it uses 10 symbols to represent all possible numbers.

Learn more about number systems at

https://brainly.com/question/14567654

#SPJ4

Triangle G H J is rotated 90 degrees about point X to form triangle S T R.

Triangle GHJ is rotated 90° about point X, resulting in triangle STR. Which congruency statement is true?
TR ≅ GJ
∠S ≅ ∠H
TS ≅ HG
∠R ≅ ∠G

Answers

The congruency statement is true TS ≅ HG. so option C is correct.

What is the congruent triangle?

Two triangles are said to be congruent if the length of the sides is equal, a measure of the angles are equal and they can be superimposed.

Given information;

Triangle G H J is rotated 90 degrees about point X to form triangle S T R.

Triangle GHJ is rotated 90° about point X, resulting in triangle STR.

Now, if any triangle is rotated 90 degrees about a point the two sides will be ≅ to each other.

So, ST = GH

RT= HJ

SR= GJ

<S= <G

<T = <H

<R=<J

The side TS ≅ HG.

Hence, option (c) is correct.

Learn more about congruent;

https://brainly.com/question/12812405

#SPJ1

Answer:

C

Step-by-step explanation:

Trust me bro

Pete drive 50 miles in 4 hours work out his average speed in miles per hour

Answers

Answer:

Pete's average speed is 12.5 miles per hour

Step-by-step explanation:

Miles per hour means how many miles were driven in 1 hour so :

50 miles in 4 hours = ? miles in 1 hour
4 ÷ x = 1

4/x = 1

x = 4

50÷4 = ?

50/4 = ?

? = 12.5

Pete's average speed is 12.5 miles per hour

Hope this helped and have a good day

Can someone help me with question 3? it geometry, gotta need an answer fast

Answers

Answer:

Step-by-step explanation:

So for the 1st one you're going to use your protractor and line it up so that BC is at 0 degrees, that way you can look at the point A to determine the angle. Now that you have the angle you're going to take your protractor and draw one point which will be where the two lines meet, line this up in the middle and then draw a point at 0 degrees and a point at the angle you found. Now draw another point at half the angle (like in the previous answer I answered). Draw a line from the middle point and connect it to all these three points (4 in total but one of them is going connected to all of them, like the point B in the image you sent).

For the second question you're going to take the angle you found in the first answer and multiply it by 2. Now start off by drawing a middle point which will connect the two other lines. Line up your protractor so that point is at bottom but in the middle (Check my photo for clarification). Now go to the value that is twice the angle and draw a point there, and also draw a point at 0 degrees) and then connect the two points to the middle point.

For question 3 you don't really need the hint, all you need is to multiply the angle by 1.5 but I'll send another image explaining what it's trying to explain.

Please help ill give whoever answers the best the brainliest answer :)))

Answers

Answer:

3rd graph from the top.

Step-by-step explanation:

It can't be the top one because the kids shoulder doesn't start at the ground.

It can't be the second or fourth because it shows him not waiting in line.

Therefore it must be the 3rd!

Answer: 3rd option

Step-by-step explanation:

Because the description says that it starts not at 0(from the kid's shoulder), the 1st and 4th options are out.

2nd option is incorrect because the kid had to stand in line so the shoulder height doesn't change(unless he was picking stuff up or something)

That leaves the 3rd option.

Which properties are necessary to claim that the two prisms are congruent? Check all that apply.


Answers

Answer:

1. The volumes are equal.

2. The base areas are equal.

3. The prisms have the same height.

Step-by-step explanation:

I don't see no answered to check all that apply but here is the answers about properties and two prisms

Evaluate the function f(x) = − x² + 10x – 6 Find f( -5)

Answers

Answer:

f (-5) = -81

Step-by-step explanation:

f (-5) = - (-5)² + 10.(-5) - 6 = -81

Writing Exercises
587. When you convert a number from decimal notation to scientific notation, how do you know if the exponent will be positive or negative?

Answers

Answer:

Therefore, we can say that the exponent will be negative if the number is less than 1 and vice versa.

Step-by-step explanation:

We are asked to explain that when we convert a decimal number to scientific notation how will we know whether the exponent is negative.

If the given number is greater than 1 then the exponent is positive and if less than 1 the exponent is negative.

please help me with this problem

Answers

Answer:

Answer is 544

Hope it helps you


What word describes the slope of the line passing through points (5, 12) and
(5,-13)?

Answers

Answer: undefined

Step-by-step explanation:

[tex]\frac{y2-y1}{x2-x1}[/tex]

[tex]\frac{-13-12}{5-5}[/tex]

[tex]\frac{-25}{0}[/tex]

= undefined

if z varies jointly with x and y when x=3, y=-1, and z=12, what is z when x=-5 and y=6

Answers

Step-by-step explanation:

z = 12 = k × 3 × -1

12 = k×-3

-4 = k

z = k× -5×6 = k×-30 = -4 × -30 = 120

What is solve -x+4=x+6

Answers

-(-1)+4=-1+6
1+4=-1+6
5=5
I hope that helped there’s the step by step to explain a little more that first x was already a negative so so to plug it in you would have to bring that negative in ground of the answer of x like so -(-1) those negative would then cancel each other out making it just 1 but if x=1 then it would be -(1) and it was make it -1 hope this helped :)

answer with steps please

Answers

Answer:

Step-by-step explanation:

Answer:  [tex]\frac{-70a^3-195^2-20a-78}{60}[/tex]  or  [tex]-\frac{7}{6} a^3-\frac{13}{4} a^2-\frac{1}{3} a-\frac{13}{10}[/tex]  

When we say subtract this from that, the value or word after from is the one which is ahead of the equation. For example, subtract 9 from 10, then the expression will be 10-9 .  [Answer is 1]

The expression that can be formed from the picture is and the solution is shown in the steps below.

[tex](\frac{a^3}{3} -\frac{3a^2}{4} -\frac{5}{2}) - (\frac{5a^2}{2} +\frac{3a^3}{2} +\frac{a}{3} -\frac{6}{5} )[/tex]           [Formed the expression]
[tex]\frac{a^3}{3} -\frac{3a^2}{4} -\frac{5}{2} - \frac{5a^2}{2} -\frac{3a^3}{2} -\frac{a}{3} +\frac{6}{5}[/tex]                 [Removed the brackets]
[tex]\frac{a^3-a}{3} +\frac{6}{5} -\frac{3a^2}{4} +\frac{-5a^2-3a^3-5}{2}[/tex]       [Brought common denominators together]
[tex]\frac{20(a^3-a)}{60} +\frac{6*12}{60} -\frac{15(3a^2)}{60} +\frac{30(-5a^2-3a^3-5)}{60}[/tex]       [LCM of 3,5,4 and 2]
[tex]\frac{20a^3-20a+72-45a^2-150a^2-90a^3-150}{60}[/tex]                  [Multiplied the numerators]
[tex]\frac{-70a^3-195^2-20a-78}{60}[/tex]                                                    [simplified]
[tex]-\frac{7}{6} a^3-\frac{13}{4} a^2-\frac{1}{3} a-\frac{13}{10}[/tex]                                        [More simplified]

I tried my best to show my steps and solve it. Thank you.

 ∩_∩
(„• ֊ •„)♡
┏━∪∪━━━━┓
    hope it helped
┗━━━━━━━┛

Which statement about the function is true?
O The function is increasing for all real values of x
where
x<-4.

Answers

Answer:

A.) The function is increasing for all real values of x where x < -4

Step-by-step explanation:

On the interval x < -4, all of the x values are increasing. Before x = -4, the line assumes a positive slope because it is heading upwards. After this interval, the function is decreasing because the slope is heading downwards.

On the interval -6 < x < -2, all of the x values are positive because they lie above the x-axis. This does not necessarily mean that all of the values are increasing.

On the interval x < -6 and x > -2, all of the x values are negative because they lie below the x-axis.

A gardener is planning to fill her garden with mulch. she plots it on a grid to plan how much she will need. the garden is in the shape of a rectangle with vertices at (3, 9) (5, 9) (3, 3) (5, 3). one bag of mulch covers 5ft^2 and costs $5.00. how much will it cost her to cover her garden?

Answers

The cost to cover the garden with mulch will be $12.00, as the area of the rectangular garden is 12 ft.².

The cost of one bag of mulch is given as $5.00.

The area covered by one bag of mulch is 5 ft.².

Therefore, the cost to cover 1 ft.² of the garden with mulch is $(5/5) = $1.00.

To find the area of the rectangular plot representing the garden, we do the follows:

The distance formula between two points (x₁, y₁) and (x₂, y₂) is,

d = √((x₂ - x₁)² + (y₂ - y₁)²).

The length of the rectangular plot between points (3, 9) and (5, 9):

Length = √((5 - 3)² + (9 - 9)²) = √(2² + 0²) = √(4) = 2.

The width of the rectangular plot between the points (3, 3) and (3, 9):

Width = √((3 - 3)² + (9 - 3)²) = √(0² + 6²) = √(36) = 6.

Therefore, the area of the rectangular plot = Length*Width = 2*6 = 12 ft.².

Therefore, the cost to cover the rectangular plot with mulch, at the cost of $1.00 per 1 ft.² = $1*12 = $12.00.

Therefore, the cost to cover the garden with mulch will be $12.00, as the area of the rectangular garden is 12 ft.².

Learn more about area of rectangles at

https://brainly.com/question/12628587

#SPJ4

How is copying segment similar to copying an angle ?

Answers

Answer:

Copying a line segment or angle is an illustration of transformation. Both are similar, because they are both rigid transformation.

When an angle is copied to a new angle, the new angle and the old angle will have the same measure

Similarly;

When a line segment is copied to a new segment, the new line segment and the old line segment will have the same measure

This type of transformation is referred to as a rigid transformation.

Hence, copying a line segment is similar to copying an angle, because they are both rigid transformations

Step-by-step explanation:


Find the circumference and the area of a circle with diameter 6 ft.

Answers

Answer:

Circumference = 18.84 ft

Area = 28.26 ft squared

Step-by-step explanation:

C(circumference) = 2 x [tex]\pi[/tex] x r(radius)

radius = 1/2 circumference

1. divide circumference by two to get the radius, which would be three ft.

2. multiply the radius by pie (3.14) and then multiply that by two and you have your circumference, which is 18.84 ft

area = [tex]\pi[/tex] x r(radius)^2

1. find the radius squared, which would be 9 because 3^2 = 3 x 3 which = 9

2. multiply 9 by pie (3.14) and you get the area which =  28.26

Victoria needs to order some new supplies for the restaurant where she works. The restaurant needs at least 673 glasses. There are currently 368 glasses. If each set on sale contains 10 glasses, write and solve an inequality which can be used to determine x, the number of sets of glasses Victoria could buy for the restaurant to have enough glasses.

Answers

Using a linear function, we have that:

The inequality is: [tex]368 + 10x \geq 673[/tex].The solution is: [tex]x \geq 31[/tex].

What is a linear function?

A linear function is modeled by:

y = mx + b

In which:

m is the slope, which is the rate of change, that is, by how much y changes when x changes by 1.b is the y-intercept, which is the value of y when x = 0, and can also be interpreted as the initial value of the function.

Since there are currently 368 glasses(y-intercept), and each set on sale comes with 10 glasses(slope), the number of glasses after x sales is given by:

y = 368 + 10x.

She needs at least 673 glasses, hence the inequality is:

[tex]y \geq 673[/tex]

[tex]368 + 10x \geq 673[/tex].

Now we solve the inequality to find the number of sets needed:

[tex]10x \geq 305[/tex]

[tex]x \geq 30.5[/tex]

As the number of sets is an integer number, the solution is:

[tex]x \geq 31[/tex].

More can be learned about linear functions at https://brainly.com/question/24808124

#SPJ1

Please help!!! What is the slope of a line perpendicular to the following line:

Answers

Answer:

b.) 2

Explanation:

Parallel slopes have same slope.

Perpendicular slopes have negatively inverse slope.

[tex]\sf Here \ given \ equation: y = -\dfrac{1}{2} x - 6[/tex]

Comparing it with slope intercept form, "y = mx + b" where m is slope.

Here parallel slope: -1/2

Perpendicular slope: -(-1/2)⁻¹ = 2

Other Questions
Classify each aqueous solution as acidic, basic, or neutral at 25 C.pH = 3.69[OH-] = 3.7 x 10-10pH = 9.04[H+] = 6.1 x 103[H] = 8.0 x 10-10[OH-] = 7.1 x 10pH = 7.00[H+] = 1.0 x 10-7 A concentration ratio measures the Group of answer choices None of the Answers are Correct. Dollar value of total industry output produced by the largest firms. Proportion of industry output produced by all firms. Proportion of industry output produced by the largest firms. Dollar value of total industry output produced by all firms. This linear function has the domain (-2, -1, 0, 1, 2). find the range or output of. h(x)=2x+3 Once you graduate from UMA, you will have an opportunity to use the skills and knowledge yourve gained to make a positive impact on clients and patients. For this discussion, conduct a job search to explore billing and coding specialist job opportunities available in your town, city, or state. You can use employment-related search engines like Indeed, SimplyHired, or Glassdoor. Use your insights to help you answer the discussion prompts. 2. Initial Post: Create a new thread and answer all three parts of the initial prompt below Introduce yourself to your instructor and classmates. Be sure to include the following: Reflect on your job search. Describe one (1) job opportunity that interests you. Explain why the job opportunity you described above interest you. Describe one step you plan to take to help you prepare more for your future career. HELP HELP HELP Just a quick question for 100 points! :) PLS HELP 1HOUR LEFT Select the expression that is modeled on the number line. A number line from zero to one partitioned into sixths. There are four hops beginning at one sixth and ending at five sixths one sixth plus five sixths one sixth plus four sixths five sixths minus one sixth five sixths minus four sixths i need help with this ASAP i've been stuck on this question for a while Construct a confidence interval for p1-p2 at the given level of confidence.x1=30, n1=235, x2=39, n2=294, 90 % confidence In the laboratory, a student dilutes 13.5 mL of a 11.6 M hydroiodic acid solution to a total volume of 100.0 mL. What is the concentration of the diluted solution There are 8 flavors of cupcakes on the dessert menu. You are asked to choose 3 flavors of cupcakes out of the 10 on the menu. How many differnet combinations of 3 cupcakes are there if order does not matter What is the approximate value of x in the diagram below? (Hint: You will needto use one of the trigonometric ratios given in the table.) a) If x and 125 are adjacent angles in linear pair, find x [with proper process] During the two hours of the morning rush from 8 a.m. to 10 a.m., 100 customers per hour arrive at the coffee shop. The coffee shop has a capacity of 0.8 minutes per customers. What is the number of customers waiting at 10 a.m. A car moves with an average speed of 75 kmh^-1 from town P to town Q in 2 hours. By using information, you may calculate the distance between two towns. state a derived quantity and its S.I unit What reason represents the same ratio as for every 12 apples, 9 of them are green?4: 36: 89:163: 4 find x and y please help i dont understand this ! During which time period was Earth a ball of hot liquid? The smallest type of internal boundary in the United States is a(n) __________ boundary. A. county B. city C. intracity D. state Please select the be Please help me( ` )The following diagram shows a quadrant OPQ in the sector ORST with centre O and a radius of 21 cm. Given Q is the midpoint of OR, calculate the area, in cm, of the shaded region. A 285.025B 308.045 C 365.075 D 410.025 Sarah publicly agrees with her seventh-grade classmates that parents should allow 13-year-olds to date. Later that day, she writes in her diary that she actually believes parents should prohibit kids from dating until they are at least 15 years old. Sarah's public conformity to her classmates' opinion best illustrates to power of: